LSAT and Law School Admissions Forum

Get expert LSAT preparation and law school admissions advice from PowerScore Test Preparation.

User avatar
 Dave Killoran
PowerScore Staff
  • PowerScore Staff
  • Posts: 5852
  • Joined: Mar 25, 2011
|
#27097
Complete Question Explanation
(The complete setup for this game can be found here: lsat/viewtopic.php?f=325&t=6210)

The correct answer choice is (B)

The condition in the question stem establishes the following setup:
June 98_M12_game#1_L6_explanations_game#1_#2_diagram_1.png
The question then proceeds to ask for the variable that could be replaced by K without creating a violation.

Answer choice (A) is incorrect because K cannot substitute for any member of the planting committee; if K is a member without the presence of J, the third rule would be violated.

Answer choice (B) is the correct answer choice. Removing G from the trails committee and replacing it with K would not cause any violations.

Answer choice (C) is incorrect because from the last rule the two committees must have at least one member in common, and H is the only member in common.

Answer choice (D) is incorrect because K cannot substitute for any member of the planting committee; if K is a member without the presence of J, the third rule would be violated.

Answer choice (E) is incorrect because K cannot substitute for any member of the planting committee; if K is a member without the presence of J, the third rule would be violated. In addition, M must appear on at least one of the committees and so removing M would violate the fourth rule.
You do not have the required permissions to view the files attached to this post.

Get the most out of your LSAT Prep Plus subscription.

Analyze and track your performance with our Testing and Analytics Package.